LSAT and Law School Admissions Forum

Get expert LSAT preparation and law school admissions advice from PowerScore Test Preparation.

 Jeremy Press
PowerScore Staff
  • PowerScore Staff
  • Posts: 1000
  • Joined: Jun 12, 2017
|
#74847
Complete Question Explanation

Must Be True, PR. The correct answer choice is B.

The question stem calls for an answer choice that follows directly from the principle that is stated in the passage. Thus, we must find an answer we can be certain about based on the given principle. Any uncertainty renders an answer choice incorrect.

Kay's principle has two parts, each of which is a conditional statement that can be diagrammed:

1. Disagree with all other candidates on even more issues important to her ----> Acceptable to vote for candidate whose opinions differ on at least one issue important to her

To apply this principle (i.e., to activate the sufficient condition), there must be a candidate with whom Kay disagrees on at least one issue important to her, plus other candidates with whom Kay disagrees on two or more such issues.

There is only one issue important to Kay in this election, and Medina shares her opinion on that issue (so as far as we know, there is no disagreement between Kay and Medina). The first part of Kay's principle is therefore inapplicable in this election. We cannot say for whom it would be acceptable for Kay to vote, because there is no rule about when (or whether) it is acceptable for Kay to vote for a candidate with whom she agrees on all issues important to her.

2. NOT disagree with all other candidates on even more issues important to her -----> NOT acceptable to vote for candidate whose opinions differ on at least one issue important to her

The second part of the principle applies, when looking at whether it is acceptable for Kay to vote for Legrand or Norton. Legrand and Norton are candidates with whom Kay disagrees on one issue that is important to her. Because Kay does not disagree with Medina on even more issues (in fact, Kay does not disagree with Medina on ANY issues important to her), the principle therefore allows us to say with certainty that it is unacceptable for Kay to vote for Legrand or Norton. This is our prephrase.

Answer Choice (A): This answer choice is incorrect for two reasons. As noted above, the principle does not allow us to conclude with certainty that it is acceptable for Kay to vote for Medina. The principle also requires us to conclude that it is unacceptable for Kay to vote for Legrand.

Answer Choice (B): This is the correct answer choice. This answer choice fits our prephrase, and, for the reasons stated above, can be concluded with certainty from the second part of Kay's principle. Notice that the "only" in the answer choice is valid, because the principle does not make it unacceptable for Kay to vote for Medina (though we cannot be absolutely certain that it is acceptable for Kay to vote for Medina).

Answer Choice (C): As noted above, we cannot know whether it is unacceptable (or acceptable) for Kay to vote for Medina.

Answer Choice (D): This answer choice is incorrect, both because we cannot conclude it is unacceptable for Kay to vote for Medina, and because we know that it is unacceptable for Kay to vote for either of Legrand or Norton.

Answer Choice (E): This answer choice is incorrect, because it is unacceptable for Kay to vote for either of Legrand or Norton.
 allisonellen7
  • Posts: 32
  • Joined: Sep 13, 2014
|
#17037
I don't understand why the only unacceptable courses of action are for Kay to vote for Norton and for her to vote for Legrand. Obviously this makes the most intuitive sense because she disagrees with them on the only issue important to her, but according to what the prompt actually said, "it is acceptable for me to vote for a candidate whose opinions differ from mine on at least one issue important to me whenever I disagree with each of the other candidates on even more such issues; it is otherwise unacceptable to vote for that candidate," so I thought it was unacceptable for Kay to vote for any of the candidates, since she didn't disagree with any of them on any issues that were important to her. Obviously, this principle wouldn't really make sense; most people would want to vote for someone they didn't disagree with on any of the issues important to them if each of the other candidates disagreed with them on some important issue; however, I have learned with logical reasoning to go by the specifics of the prompt, not on common sense, and it lead me wrong! Thanks in advance!
 Emily Haney-Caron
PowerScore Staff
  • PowerScore Staff
  • Posts: 577
  • Joined: Jan 12, 2012
|
#17043
Hi Allison Ellen,

The wording here is a bit tricky, and I think rearranging the sentence you highlighted might help. Essentially, the prompt is saying, "It is unacceptable for me to vote for a candidate whose opinions differ from mine on at least one issue important to me UNLESS I disagree with each of the other candidates on even more such issues."
The meaning is exactly the same; we've just reordered the clauses. Following that rule, it is unacceptable for her to vote for N or L UNLESS she disagrees with each of the other candidates on more than one issue important to her (which obviously is not the case here, since there is only a single issue important to her).

I think you just missed the part where the prompt says it is acceptable...WHENEVER I disagree with each of the other candidates on even more such issues.

Does that help?
 allisonellen7
  • Posts: 32
  • Joined: Sep 13, 2014
|
#17044
Yes, that makes a lot of sense! Thanks so much!
 hwkim93
  • Posts: 5
  • Joined: Aug 22, 2018
|
#49942
Hello.

I had a question regarding why it isn't unacceptable for Kay to vote for Melinda. Is it because the stimulus doesn't say anything about how Kay should vote on a candidate who shares Kay's opinion on all of the issues that are important to her?

Thank you!
 James Finch
PowerScore Staff
  • PowerScore Staff
  • Posts: 943
  • Joined: Sep 06, 2017
|
#50269
Hi HW,

Medina is the only candidate that Kay could potentially accept voting for, but because we don't know how her plan works when she agrees with a candidate on all important issues, we don't know for sure if voting for him would be acceptable or not. What we do know is that voting for the other two would be unacceptable, according to the logic of the stimulus. Kay doesn't have to vote for Medina, and she could always choose not to, but she for sure can't vote for the other two candidates. So we need to prephrase this as the one thing we can infer from the stimulus:
"The only unacceptable courses of action are to vote for Legrand or Norton"

Ultimately this gets us right to the correct answer choice, (B).

Hope this clears things up!
 hwkim93
  • Posts: 5
  • Joined: Aug 22, 2018
|
#54100
Thank you for the explanation!
 astroworld
  • Posts: 8
  • Joined: Dec 05, 2019
|
#74775
Dear Powerscore,

First of all, I would like to thank you for providing answers in such great detail, it is really helpful to be able to ask these questions as I am currently self-studying.

I have been struggling with this question for quite some time, there were few things that still weren't clear to me.

The stimulus says that there is only one issue important to Kay and only Medina shares her opinion. So hypothesizing that she disagrees with Medina, there is only one issue that she is in disagreement with Medina. Therefore, even if Kay has the least number of disagreement on an issue which is 1 for Legrand or Norton, it would never be acceptable to vote for neither of them because the number of disagreement with Medina cannot be more than 1. And since, we don't know how many disagreement there would be with Legrand and Norton, we cannot conclude whether Kay would find acceptable to vote for Medina because they are both possible.

My apologies for writing in such length...Would you be able to tell me if there is anything I am missing with my reasoning?

Thank you!
 Jeremy Press
PowerScore Staff
  • PowerScore Staff
  • Posts: 1000
  • Joined: Jun 12, 2017
|
#74806
Hi astroworld,

Check out the complete question explanation I have posted at the top of thread, and let us know if this resolves your questions. If not, let us know about anything that remains unclear, because we'd love to discuss further!

Jeremy
 astroworld
  • Posts: 8
  • Joined: Dec 05, 2019
|
#74817
So, on the last part of the stimulus: "There is only one issue important to Kay, and only Medina shares her opinion on that issue" are we assuming that Kay is disagreeing on that issue with Legrand and Norton?

Because I thought, since it didn't mention anything, there would be a possibility which Legrand and Norton just didn't have any opinions on that issue at all.

Also, where can I find these full explanations, are they posted on this forum?

Thanks!

Get the most out of your LSAT Prep Plus subscription.

Analyze and track your performance with our Testing and Analytics Package.